Prove that there exists a row or a column of the chessboard which contains at least √n distinct numbers.

The name of the pictureThe name of the pictureThe name of the pictureClash Royale CLAN TAG#URR8PPP











up vote
5
down vote

favorite
1













On each cell of an $n times n$ chessboard, we write a number from $1, 2, 3, .
. . , n$
in such a way that each number appears exactly $n$ times. Prove
that there exists a row or a column of the chessboard which contains
at least $sqrtn$ distinct numbers.




This question is dying for a proof by contradiction, but I can't seem to show how. When I searched through the Internet, there is the following hint:




Assume each row and each column has less than $sqrtn$ distinct numbers in
it. For each row or column, consider the number of distinct numbers in
it.




I feel like double counting is involved in the process, but what to count in two different ways? Also I can prove the statement with the probabilistic method but I would really like to learn the double counting approach.










share|cite|improve this question























  • For reference, and to help others, could you post your probability-based proof, either as part of your question, or perhaps as an answer? I would very much like to see it. Thanks.
    – quasi
    2 hours ago











  • @quasi I am posting it now
    – user2694307
    8 mins ago















up vote
5
down vote

favorite
1













On each cell of an $n times n$ chessboard, we write a number from $1, 2, 3, .
. . , n$
in such a way that each number appears exactly $n$ times. Prove
that there exists a row or a column of the chessboard which contains
at least $sqrtn$ distinct numbers.




This question is dying for a proof by contradiction, but I can't seem to show how. When I searched through the Internet, there is the following hint:




Assume each row and each column has less than $sqrtn$ distinct numbers in
it. For each row or column, consider the number of distinct numbers in
it.




I feel like double counting is involved in the process, but what to count in two different ways? Also I can prove the statement with the probabilistic method but I would really like to learn the double counting approach.










share|cite|improve this question























  • For reference, and to help others, could you post your probability-based proof, either as part of your question, or perhaps as an answer? I would very much like to see it. Thanks.
    – quasi
    2 hours ago











  • @quasi I am posting it now
    – user2694307
    8 mins ago













up vote
5
down vote

favorite
1









up vote
5
down vote

favorite
1






1






On each cell of an $n times n$ chessboard, we write a number from $1, 2, 3, .
. . , n$
in such a way that each number appears exactly $n$ times. Prove
that there exists a row or a column of the chessboard which contains
at least $sqrtn$ distinct numbers.




This question is dying for a proof by contradiction, but I can't seem to show how. When I searched through the Internet, there is the following hint:




Assume each row and each column has less than $sqrtn$ distinct numbers in
it. For each row or column, consider the number of distinct numbers in
it.




I feel like double counting is involved in the process, but what to count in two different ways? Also I can prove the statement with the probabilistic method but I would really like to learn the double counting approach.










share|cite|improve this question
















On each cell of an $n times n$ chessboard, we write a number from $1, 2, 3, .
. . , n$
in such a way that each number appears exactly $n$ times. Prove
that there exists a row or a column of the chessboard which contains
at least $sqrtn$ distinct numbers.




This question is dying for a proof by contradiction, but I can't seem to show how. When I searched through the Internet, there is the following hint:




Assume each row and each column has less than $sqrtn$ distinct numbers in
it. For each row or column, consider the number of distinct numbers in
it.




I feel like double counting is involved in the process, but what to count in two different ways? Also I can prove the statement with the probabilistic method but I would really like to learn the double counting approach.







combinatorics






share|cite|improve this question















share|cite|improve this question













share|cite|improve this question




share|cite|improve this question








edited 3 hours ago









greedoid

31.2k94287




31.2k94287










asked 5 hours ago









user2694307

163210




163210











  • For reference, and to help others, could you post your probability-based proof, either as part of your question, or perhaps as an answer? I would very much like to see it. Thanks.
    – quasi
    2 hours ago











  • @quasi I am posting it now
    – user2694307
    8 mins ago

















  • For reference, and to help others, could you post your probability-based proof, either as part of your question, or perhaps as an answer? I would very much like to see it. Thanks.
    – quasi
    2 hours ago











  • @quasi I am posting it now
    – user2694307
    8 mins ago
















For reference, and to help others, could you post your probability-based proof, either as part of your question, or perhaps as an answer? I would very much like to see it. Thanks.
– quasi
2 hours ago





For reference, and to help others, could you post your probability-based proof, either as part of your question, or perhaps as an answer? I would very much like to see it. Thanks.
– quasi
2 hours ago













@quasi I am posting it now
– user2694307
8 mins ago





@quasi I am posting it now
– user2694307
8 mins ago











1 Answer
1






active

oldest

votes

















up vote
5
down vote



accepted










So we have $C_1,...C_n$ columns and $R_1,...,R_n$ rows. Each of these we call a line. So we have $2n$ lines $L_1,L_2,...,L_2n$ and suppose that each line has less than $sqrtn$ different numbers ($d(L_i)<sqrtn$).



Suppose number $kin1,2,...,n$ appears in $c_k$ columns and $r_k$ rows. Then we see that $$r_k+c_k geq 2sqrtn$$



(Imagine a square $sqrtntimes sqrtn$ filled with the same number. ''Clearly'' at this configuration is the minimum for $r_k+c_k$).



So we have: $$2ncdot sqrtn>sum _i=1^2n d(L_i) = sum_k=1^n (r_k+c_k)geq ncdot (2sqrtn)$$



A contradiction.






share|cite|improve this answer






















  • Can you explain the claim $r_k+c_kge 2sqrtn$?
    – quasi
    3 hours ago











  • Imagine a square $sqrtntimes sqrtn$ filed with the same number. ''Clearly'' at this configuration is the minimum for $r_k+c_k$
    – greedoid
    3 hours ago







  • 1




    Yes, I see. I think that mini-argument should be included in the proof.
    – quasi
    3 hours ago






  • 1




    Nice proof! (+1).
    – quasi
    3 hours ago










  • I actually made use of the mini square argument in the probabilistic proof. Seeing that it also works here is great. Thanks a lot.
    – user2694307
    2 hours ago










Your Answer




StackExchange.ifUsing("editor", function ()
return StackExchange.using("mathjaxEditing", function ()
StackExchange.MarkdownEditor.creationCallbacks.add(function (editor, postfix)
StackExchange.mathjaxEditing.prepareWmdForMathJax(editor, postfix, [["$", "$"], ["\\(","\\)"]]);
);
);
, "mathjax-editing");

StackExchange.ready(function()
var channelOptions =
tags: "".split(" "),
id: "69"
;
initTagRenderer("".split(" "), "".split(" "), channelOptions);

StackExchange.using("externalEditor", function()
// Have to fire editor after snippets, if snippets enabled
if (StackExchange.settings.snippets.snippetsEnabled)
StackExchange.using("snippets", function()
createEditor();
);

else
createEditor();

);

function createEditor()
StackExchange.prepareEditor(
heartbeatType: 'answer',
convertImagesToLinks: true,
noModals: false,
showLowRepImageUploadWarning: true,
reputationToPostImages: 10,
bindNavPrevention: true,
postfix: "",
noCode: true, onDemand: true,
discardSelector: ".discard-answer"
,immediatelyShowMarkdownHelp:true
);



);













 

draft saved


draft discarded


















StackExchange.ready(
function ()
StackExchange.openid.initPostLogin('.new-post-login', 'https%3a%2f%2fmath.stackexchange.com%2fquestions%2f2950325%2fprove-that-there-exists-a-row-or-a-column-of-the-chessboard-which-contains-at-le%23new-answer', 'question_page');

);

Post as a guest






























1 Answer
1






active

oldest

votes








1 Answer
1






active

oldest

votes









active

oldest

votes






active

oldest

votes








up vote
5
down vote



accepted










So we have $C_1,...C_n$ columns and $R_1,...,R_n$ rows. Each of these we call a line. So we have $2n$ lines $L_1,L_2,...,L_2n$ and suppose that each line has less than $sqrtn$ different numbers ($d(L_i)<sqrtn$).



Suppose number $kin1,2,...,n$ appears in $c_k$ columns and $r_k$ rows. Then we see that $$r_k+c_k geq 2sqrtn$$



(Imagine a square $sqrtntimes sqrtn$ filled with the same number. ''Clearly'' at this configuration is the minimum for $r_k+c_k$).



So we have: $$2ncdot sqrtn>sum _i=1^2n d(L_i) = sum_k=1^n (r_k+c_k)geq ncdot (2sqrtn)$$



A contradiction.






share|cite|improve this answer






















  • Can you explain the claim $r_k+c_kge 2sqrtn$?
    – quasi
    3 hours ago











  • Imagine a square $sqrtntimes sqrtn$ filed with the same number. ''Clearly'' at this configuration is the minimum for $r_k+c_k$
    – greedoid
    3 hours ago







  • 1




    Yes, I see. I think that mini-argument should be included in the proof.
    – quasi
    3 hours ago






  • 1




    Nice proof! (+1).
    – quasi
    3 hours ago










  • I actually made use of the mini square argument in the probabilistic proof. Seeing that it also works here is great. Thanks a lot.
    – user2694307
    2 hours ago














up vote
5
down vote



accepted










So we have $C_1,...C_n$ columns and $R_1,...,R_n$ rows. Each of these we call a line. So we have $2n$ lines $L_1,L_2,...,L_2n$ and suppose that each line has less than $sqrtn$ different numbers ($d(L_i)<sqrtn$).



Suppose number $kin1,2,...,n$ appears in $c_k$ columns and $r_k$ rows. Then we see that $$r_k+c_k geq 2sqrtn$$



(Imagine a square $sqrtntimes sqrtn$ filled with the same number. ''Clearly'' at this configuration is the minimum for $r_k+c_k$).



So we have: $$2ncdot sqrtn>sum _i=1^2n d(L_i) = sum_k=1^n (r_k+c_k)geq ncdot (2sqrtn)$$



A contradiction.






share|cite|improve this answer






















  • Can you explain the claim $r_k+c_kge 2sqrtn$?
    – quasi
    3 hours ago











  • Imagine a square $sqrtntimes sqrtn$ filed with the same number. ''Clearly'' at this configuration is the minimum for $r_k+c_k$
    – greedoid
    3 hours ago







  • 1




    Yes, I see. I think that mini-argument should be included in the proof.
    – quasi
    3 hours ago






  • 1




    Nice proof! (+1).
    – quasi
    3 hours ago










  • I actually made use of the mini square argument in the probabilistic proof. Seeing that it also works here is great. Thanks a lot.
    – user2694307
    2 hours ago












up vote
5
down vote



accepted







up vote
5
down vote



accepted






So we have $C_1,...C_n$ columns and $R_1,...,R_n$ rows. Each of these we call a line. So we have $2n$ lines $L_1,L_2,...,L_2n$ and suppose that each line has less than $sqrtn$ different numbers ($d(L_i)<sqrtn$).



Suppose number $kin1,2,...,n$ appears in $c_k$ columns and $r_k$ rows. Then we see that $$r_k+c_k geq 2sqrtn$$



(Imagine a square $sqrtntimes sqrtn$ filled with the same number. ''Clearly'' at this configuration is the minimum for $r_k+c_k$).



So we have: $$2ncdot sqrtn>sum _i=1^2n d(L_i) = sum_k=1^n (r_k+c_k)geq ncdot (2sqrtn)$$



A contradiction.






share|cite|improve this answer














So we have $C_1,...C_n$ columns and $R_1,...,R_n$ rows. Each of these we call a line. So we have $2n$ lines $L_1,L_2,...,L_2n$ and suppose that each line has less than $sqrtn$ different numbers ($d(L_i)<sqrtn$).



Suppose number $kin1,2,...,n$ appears in $c_k$ columns and $r_k$ rows. Then we see that $$r_k+c_k geq 2sqrtn$$



(Imagine a square $sqrtntimes sqrtn$ filled with the same number. ''Clearly'' at this configuration is the minimum for $r_k+c_k$).



So we have: $$2ncdot sqrtn>sum _i=1^2n d(L_i) = sum_k=1^n (r_k+c_k)geq ncdot (2sqrtn)$$



A contradiction.







share|cite|improve this answer














share|cite|improve this answer



share|cite|improve this answer








edited 1 hour ago









Avraham

2,4771131




2,4771131










answered 4 hours ago









greedoid

31.2k94287




31.2k94287











  • Can you explain the claim $r_k+c_kge 2sqrtn$?
    – quasi
    3 hours ago











  • Imagine a square $sqrtntimes sqrtn$ filed with the same number. ''Clearly'' at this configuration is the minimum for $r_k+c_k$
    – greedoid
    3 hours ago







  • 1




    Yes, I see. I think that mini-argument should be included in the proof.
    – quasi
    3 hours ago






  • 1




    Nice proof! (+1).
    – quasi
    3 hours ago










  • I actually made use of the mini square argument in the probabilistic proof. Seeing that it also works here is great. Thanks a lot.
    – user2694307
    2 hours ago
















  • Can you explain the claim $r_k+c_kge 2sqrtn$?
    – quasi
    3 hours ago











  • Imagine a square $sqrtntimes sqrtn$ filed with the same number. ''Clearly'' at this configuration is the minimum for $r_k+c_k$
    – greedoid
    3 hours ago







  • 1




    Yes, I see. I think that mini-argument should be included in the proof.
    – quasi
    3 hours ago






  • 1




    Nice proof! (+1).
    – quasi
    3 hours ago










  • I actually made use of the mini square argument in the probabilistic proof. Seeing that it also works here is great. Thanks a lot.
    – user2694307
    2 hours ago















Can you explain the claim $r_k+c_kge 2sqrtn$?
– quasi
3 hours ago





Can you explain the claim $r_k+c_kge 2sqrtn$?
– quasi
3 hours ago













Imagine a square $sqrtntimes sqrtn$ filed with the same number. ''Clearly'' at this configuration is the minimum for $r_k+c_k$
– greedoid
3 hours ago





Imagine a square $sqrtntimes sqrtn$ filed with the same number. ''Clearly'' at this configuration is the minimum for $r_k+c_k$
– greedoid
3 hours ago





1




1




Yes, I see. I think that mini-argument should be included in the proof.
– quasi
3 hours ago




Yes, I see. I think that mini-argument should be included in the proof.
– quasi
3 hours ago




1




1




Nice proof! (+1).
– quasi
3 hours ago




Nice proof! (+1).
– quasi
3 hours ago












I actually made use of the mini square argument in the probabilistic proof. Seeing that it also works here is great. Thanks a lot.
– user2694307
2 hours ago




I actually made use of the mini square argument in the probabilistic proof. Seeing that it also works here is great. Thanks a lot.
– user2694307
2 hours ago

















 

draft saved


draft discarded















































 


draft saved


draft discarded














StackExchange.ready(
function ()
StackExchange.openid.initPostLogin('.new-post-login', 'https%3a%2f%2fmath.stackexchange.com%2fquestions%2f2950325%2fprove-that-there-exists-a-row-or-a-column-of-the-chessboard-which-contains-at-le%23new-answer', 'question_page');

);

Post as a guest













































































Comments

Popular posts from this blog

Long meetings (6-7 hours a day): Being “babysat” by supervisor

Is the Concept of Multiple Fantasy Races Scientifically Flawed? [closed]

Confectionery